The amount that Za Kingdom charges for each pizza is $8.
How to determine the amount charged by Za kingdomTo determine the amount charged by Za Kingdom, we first note that the cost of each pizza plus the delivery fee is $21.
Assuming the basic price to be $8 and the delivery fee of $13, we have the sum of $21. So, for 3 pizzas, we will have 8 times 3 = 24 + 13 = 37
Also, the amount for 10 pizzas will be 8 times 10 = 80 + 13 = 93. So, the price of each pizza is $8.
Learn more about prices here:
https://brainly.com/question/1153322
#SPJ1
The values in the table show the relationship between times
measured in seconds and distances measured in meters.
Seconds
5
10
15
20
Meters
20
40
60
80
What is the unit rate, In meters per second, of the relationship
shown in the table?
Enter your answer in the box.
meters per second
the unit rate is 4 meters per second, meaning that for every second that passes, the distance increases by 4 meters.
The table shows the relationship between time and distance time is measured in seconds and distance is measured in meters.
The table provides us with the data points of time and distance allows us to examine the relationship between these two variables.
The table as time increases the distance also increases.
This suggests that there is a positive relationship between time and distance.
The unit rate of the relationship tells us how much distance changes for every unit of time.
The unit rate is 4 meters per second meaning that for every second that passes, the distance increases by 4 meters.
The relationship shown in the table is not necessarily a linear relationship.
A linear relationship would mean that the distance changes by the same amount for each unit of time.
The increase in distance is not constant over time.
From 5 seconds to 10 seconds the distance increases by 20 meters but from 10 seconds to 15 seconds the distance increases by 20 meters again and from 15 seconds to 20 seconds the distance increases by 20 meters yet again.
The table provides us with a snapshot of the relationship between time and distance and by examining the data we can identify patterns and relationships that can help us understand the behavior of these variables.
For similar questions on unit rate
https://brainly.com/question/4895463
#SPJ11
Solve for measure of angle a.
41
03
29°
a = [ ? ]°
If two chords intersect inside a circle:
- bec
The measure of angle a is 35°
How to solve for measure of angle a?The chord of a circle is the line segment joining any two points on the circumference of the circle.
The diameter is the longest chord of a circle which passes through the center of the circle.
For chords that meet inside of a circle, the measure of the angle formed is one-half the sum of the measure of the arcs intercepted by the angle and its vertical angle.
Using the above concept (That is the given hint in the question), we can say:
∠a = (29 + 41)/2
∠a = 70/2
∠a = 35°
Therefore, the measure of angle a is 35°
Learn more about arcs on:
brainly.com/question/31105144
#SPJ1
Complete Question
Check attached image
You and some friends buy hamburgers
and milkshakes for lunch. A hamburger
costs $1.50 and a milkshake costs $2.00.
The total bill for 12 items is $21.50.
How many hamburgers and how many
milkshakes did your group buy?
(Please hurry and show work.)
Answer:
5 burgers and 7 milkshakes
Step-by-step explanation:
1.5x5=7.5
2x7=14
7.5+14=21.5
Simplify
| 2(11-3)/2|^2
Answer:
To simplify the expression |2(11-3)/2|^2, we follow the order of operations (parentheses, exponents, multiplication and division from left to right, and addition and subtraction from left to right).
First, simplify the expression inside the absolute value:
2(11-3)/2 = 2(8)/2 = 16/2 = 8
Now, simplify the absolute value of 8, which is simply the magnitude of 8:
|8| = 8
Finally, square the result:
8^2 = 8 * 8 = 64
Therefore, the simplified value of |2(11-3)/2|^2 is 64.
Step-by-step explanation:
Bernard can save $315 per month that he puts into a savings account
earning 7% annual interest. How much will he have saved after 4
years? show your work
After 4 years of saving $315 per month at 7% annual interest rate, Bernard will have saved approximately $10,864.11.
To calculate the amount that Bernard will have saved after 4 years, we can use the formula for compound interest:
A = P(1 + r/n[tex])^{(nt)[/tex]
Where:
A = the amount of money Bernard will have saved after 4 years
P = the amount of money Bernard is putting into savings each month = $315
r = the annual interest rate = 7% = 0.07
n = the number of times interest is compounded per year (monthly compounding in this case) = 12
t = the number of years = 4
Substituting the given values into the formula, we get:
A = 315(1 + 0.07/12[tex])^{(12\times 4)[/tex]
A = 315(1.00583333[tex])^{48[/tex]
A = 315(1.31564)
A = $10,864.11
Therefore, after 4 years of saving $315 per month at 7% annual interest rate, Bernard will have saved approximately $10,864.11.
for such more question on interest rate
https://brainly.com/question/29451175
#SPJ11
1. The list of rational numbers below are
ordered from greatest to least.
15, 6.7, , 100%, -55%,
Which of the following numbers could NOT be
the missing value in the list above?
A. -80%
C.-2.6
B. -1
D. - 180%
-2.5
Answer:
The list of rational numbers below are ordered from greatest to least.
15, 6.7, , 100%, -55%,
To determine which of the following numbers could NOT be the missing value in the list above, we need to determine the correct order of the numbers in the list.
First, we can convert the percentages to decimals:
100% = 1
-55% = -0.55
Now we can order the numbers from greatest to least:
15, 6.7, 1, -0.55, x
We know that x is a rational number, so it can be expressed as a fraction. We can eliminate answer choices C and D because they are not rational numbers.
Next, we can compare x to the other numbers in the list. We know that x must be less than 6.7, so we can eliminate answer choice A.
That leaves us with answer choice B, -2.5. This number could be the missing value in the list, so the answer is B.
car cost #450,000 and its being sold for #540,000. find the profit as a percentage of the cost price
The profit as a percentage of the cost price is 20%
Finding the profit as a percentage of the cost priceFrom the question, we have the following parameters that can be used in our computation:
Car cost #450,000It;s being sold for #540,000.The profit is the difference beyween the selling price and the coste price
So, we have
Profit = 540000 - 450000
Evaluate
Profit = 90000
Next, we have
Percentage = Profit/Cost price * 100
This gives
Percentage = 90000/450000* 100
Evaluate
Percentage = 20%
Hence, teh percentage is 20%
Read mroe about percentage at
https://brainly.com/question/24877689
#SPJ1
Answer:
Step-by-step explanation:
To find the profit as a percentage of the cost price, we need to calculate the profit first.
Profit = Selling price - Cost price
Profit = #540,000 - #450,000
Profit = #90,000
Now we can calculate the profit percentage as follows:
Profit percentage = (Profit / Cost price) x 100
Profit percentage = (#90,000 / #450,000) x 100
Profit percentage = 20%
Therefore, the profit as a percentage of the cost price is 20%.
Find all the zeros of the polynomial p(x)=x^3+x^2+8x-60
--------------------------
Enter the zeros separated by commas. Enter exact values, not decimal approximations, using square roots as needed.
The factorization of p(x) is p(x) = (x + 5)^2 (x - 3).
There are different methods to find the zeros of a polynomial, but one common approach is to use the Rational Root Theorem to test possible rational roots and then use polynomial division to factor out the roots found.
The Rational Root Theorem states that if a polynomial with integer coefficients has a rational root p/q (where p and q are integers with no common factors other than 1), then p must divide the constant term and q must divide the leading coefficient.
In the case of p(x) = x³ + x² + 8x - 60, the constant term is -60 and the leading coefficient is 1, so the possible rational roots are of the form p/q, where p is a factor of -60 and q is a factor of 1. Therefore, the possible rational roots are ±1, ±2, ±3, ±4, ±5, ±6, ±10, ±12, ±15, ±20, ±30, ±60.
Since the remainder is not zero, x = 1 is not a root of p(x). We can repeat this process with each possible root until we find all the actual roots.
Using this method, we find that the zeros of p(x) are:
x = -5 (with multiplicity 2) and x = 3.
Therefore, the factorization of p(x) is:
p(x) = (x + 5)² (x - 3)
To know more about factorization follow
https://brainly.com/question/11447331
#SPJ1
A farmer of a large apple orchard would like to estimate the true mean number of suitable apples produced per tree. He selects a random sample of 40 trees from his large orchard and determines with 95% confidence that the true mean number of suitable apples produced per tree is between 375 and 520. Which of these statements is a correct interpretation of the confidence level?
Approximately 95% of the trees in the orchard contain between 375 and 520 suitable apples.
There is a 95% probability that the true mean number of apples per tree is between 375 and 520.
If many random samples of size 40 are selected from the population of all trees in this large orchard, approximately 95% of the sample means will be between 375 and 520 apples.
If many random samples of size 40 are selected from the population of all trees in this large orchard, about 95% of the intervals would capture the true mean number of suitable apples produced per tree.
Answer:
The option that the margin of error accounts for in the given interval is sampling variation.
Step-by-step explanation:
Post Test: Polygons 4 Given: ΔMNO Prove: The medians of ΔMNO are concurrent. Given triangle of sides M (x1, y1), N (x2, y2) and O (x3, y3) point to the same triangle on the graph of x-axis and y-axis, with M prime (0, 0), O prime (2t, 0) and N prime (2r, 2s) has a midpoint s formed by connecting Prime PQR Proof: Statements Reasons 1. ∆MNO Given 2. Use rigid transformations to transform ΔMNO to the congruent ΔM'N'O' so that M' is at the origin and M'O' lies on the x-axis in the positive direction. Rigid transformations result in congruent shapes. 3. Any property that is true for ΔM'N'O' will also be true for ΔMNO. Definition of congruence 4. Let r, s, and t be real numbers such that the vertices of ΔM'N'O' are M'(0,0), N'(2r,2s), and O'(2t,0). Defining constants 5. Let P', Q', and R' be the midpoints of M'N', N'O', and M'O', respectively. Defining points 6. P' = (r,s); Q' = (r + t,s); R' = (t,0) ? 7. Definition of slope 8. Applying point-slope formula 9. and intersect at point S . Algebra 10. Point S lies on . The coordinates of S satisfy the equation of . 11. All three lines share point S. Follows from 9 and 10 12. The medians are concurrent. Definition of concurrent lines Which reason completes the proof for step 6? A. Definition of midsegment B. Definition of slope C. Definition of midpoint D. Definition of median
A reason that completes the proof for step 6 include the following: A. Definition of midsegment.
What is a midsegment?In Mathematics and Geometry, a midsegment can be defined as a type of line segment that is typically used for connecting the midpoints of two (2) sides of a triangle.
In order to prove that a midsegment is parallel to the median, it is imperative that the definition of the midsegment must be applied.
According to the definition of the midsegment, which states that a line segment connects the midpoints of the two (2) sides of a given triangle (ΔMNO), we can logically deduce the following:
P' = (r, s)
Q' = (r + t, s)
R' = (t, 0)
Read more on midsegment here: https://brainly.com/question/30991167
#SPJ1
Missing information:
The question is incomplete and the complete question is shown in the attached picture.
What is a formula for the nth term of the given sequence?
Select Explicit Formula for Sequence
The solution is, the nth term of the sequence is: an = 250 * 0.4(n-1)
To find a formula for the nth term of the sequence 250, 100, 40, we need to first determine the pattern or rule that generates the sequence. If we divide each term by the previous term, we get the following ratios:
100/250 = 0.4
40/100 = 0.4
We can see that each term is obtained by multiplying the previous term by the common ratio of 0.4.
So, the formula for the nth term of the sequence is:
an = a1* r(n-1)
where a1= 250 is the first term, r = 0.4 is the common ratio, and n is the term number.
The solution is, the nth term of the sequence is: an = 250 * 0.4(n-1)
To learn more about sequence here:
brainly.com/question/30262438
#SPJ1
compplete question:
What is a formula for the nth term of the given sequence 250,100,40
Compare the linear relationships.
Linear equation A: y=58x
Linear equation B is partially represented by the points in this table.
x
y
12
623
13
729
14
779
Which linear equation has the greater slope? Why?
Select the option that correctly answers both questions.
Responses
Equation B has the greater slope. It has a slope of 623,
and equation A has a slope of 58,
which is smaller.
Equation B has the greater slope. It has a slope of 6 and 2 thirds textsf comma and equation A has a slope of 5 eighths textsf comma which is smaller.
Equation A has the greater slope. It has a slope of 58,
and equation B has a slope of 59,
which is smaller.
Equation A has the greater slope. It has a slope of 5 eighths textsf comma and equation B has a slope of 5 ninths textsf comma which is smaller.
Equation A has the greater slope. It has a slope of 59,
and equation B has a slope of 58,
which is smaller.
Equation A has the greater slope. It has a slope of 5 ninths textsf comma and equation B has a slope of 5 eighths textsf comma which is smaller.
Equation B has the greater slope. It has a slope of 59,
and equation A has a slope of 58,
which is smaller.
Compare the linear relationships shows that
Equation B has the greater slope. It has a slope of 106, and Equation A has a slope of 58, which is smaller.
How to compare the slopesEquation A is given as y = 58x
The slope is 58
For Equation B
Slope of Equation B = (change in y) / (change in x)
= (729 - 623) / (13 - 12)
= 106 / 1
= 106
Comparing the slopes, we can see that the slope of Equation B (106) is greater than the slope of Equation A (58)
Learn more about Linear equation at
https://brainly.com/question/2030026
#SPJ1
A football is catapulted into the air so that its height h, in metres, after t seconds is h = -4.9t² +27t +
2.4 a) How high is the football after 1 second? b) For how long is the football more than 30 m high? c)
What is the maximum height of the football?
Answer:
a] 24.5; b] 2.8; c] 39.59.
Step-by-step explanation:
a. after t=1sec:
h(1)=-4.9*1²+27*1+2.4; ⇔h(1)=24.5 [m];
b. more than 30 [m] heigh:
-4.9t²+27t+2.4≥30; ⇔ 4.9t²-27t+27.6≤0; ⇔(t-1.35)(t-4.15)≤0;
Δt≈4.15-1.35=2.8 [sec];
c. maximum height:
h'(t)=-9.7t+27; ⇒ h'(t)=0, ⇒ -9.7(t-2.78)=0; ⇒ t=2.78 [sec], then
[tex]h_{max}=h(2.78)=-4.9*2.78^2+27*2.78+2.4=39.59[m].[/tex]
If trapezoid JKLM is translated using the rule (x, y) → (x + 3, y − 3) and then translated using the rule (x, y) → (x − 1, y + 1) to create trapezoid J″K″L″M″, what is the location of L″?
Answer:
L'' = (6, - 5 )
Step-by-step explanation:
the translation rule (x, y ) → (x + 3, y - 3 ) means
add 3 to the original x- coordinate and
subtract 3 from the original y- coordinate
then
L (3, - 2 ) → L'' (3 + 3, - 2 - 3 ) → L'' (6, - 5 )
please show work I don't understand how to do this
Answer: x= 3
Step-by-step explanation:
We solve the given equation to obtain value of x
Step 1: We open the bracket:
8x-6-8=4+2x
Step 2: We transform terms with x to left hand side and constants to the right hand side and change their signs
8x-2x=4+6+8
6x=18
x=18/6
Thus, x=3
To learn more on equations,
https://brainly.com/question/14410653?referrer=searchResults
https://brainly.com/question/2956399?referrer=searchResults
Determine the equation of the circle with center
(
0
,
9
)
(0,9) containing the point
(
65
,
12
)
(
65
,12).
A circle with Centre [tex](a,b)[/tex] has the general equation:
[tex](x-a)^2 + (y-b)^2 = r^2[/tex]
where [tex]r[/tex] is the radius and [tex](a,b)[/tex] is the Centre.
Given that the Centre is at [tex](0,9)[/tex], the equation is as follows:
[tex](x-0)^2 + (y-9)^2 = r^2[/tex]
If we simplify, we get:
[tex]x^2 + (y-9)^2 = r^2[/tex]
The fact that the circle comprises the coordinates [tex](65,12)[/tex] is also provided. Therefore, we may change these values into the equation and find [tex]r[/tex]:
[tex]65^2 + (12-9)^2 = r^2[/tex]
[tex]4225 + 9 = r^2[/tex]
[tex]r^2 = 4234[/tex]
When we square the two sides, we obtain:
[tex]r = \sqrt{4234}[/tex]
The circle's equation is as follows:
[tex]x^2 + (y-9)^2 = 4234[/tex]
To know more about equation of circle
brainly.com/question/32072534
the equation T^2=A^3 shows the relationship between a planets orbital period
The orbital increased by factor [tex]2^3^/^2[/tex] .Hence, correctander is option D by equation T²=A³
The relationship between a planet’s orbital period(T) and the planet’s mean distance from the sun(A) is given by equation T²=A³
Given equation is T²=A³
The distance between the planet X and sun is represented by d
Time-period of the planet X:
T²=d³
Distance between planet Y and the sun is represented by d'
d'= 2d
T'²=d'² = (2d)³ = 8d³
On dividing (1)and (2).
T'/T = 8d³/d³
=8
=2√2
=[tex]2^3^/^2[/tex]
Hence, the orbital increased by factor [tex]2^3^/^2[/tex] .
To learn more on Equation:
https://brainly.com/question/10413253
#SPJ1
The equation T^2=A^3 shows the relationship between a planet’s orbital period, T, and the planet’s mean distance from the sun, A, in astronomical units, AU. If planet Y is twice the mean distance from the sun as planet X, by what factor is the orbital period increased? A. 2^1/3. B. 2^1/2. C. 2^2/3. D. 2^3/2
GIVING BRAINLIEST!!!!!! A homeowner is building a circular fire pit in his backyard. He plans to outline the pit with bricks and cover the space inside the pit with sand. The homeowner has decided to build the pit with a diameter of 3 feet.
In order to know how many bricks to buy, the homeowner must know the distance around the outside of the pit. Calculate both the exact distance and the approximate distance.
In order to know how much sand to buy, the homeowner must know how much space needs to be covered inside the pit. Calculate both the exact area and the approximate area.
Answer:
To calculate the distance around the outside of the circular fire pit (circumference), we can use the formula:
Circumference = π × diameter
Given that the diameter of the pit is 3 feet, we can substitute this value into the formula:
Circumference = π × 3 feet
For the exact distance, we can use the value of π (pi) as 3.14159 (approximately):
Circumference ≈ 3.14159 × 3 feet
Circumference ≈ 9.42477 feet
For the approximate distance, we can round π to 3:
Circumference ≈ 3 × 3 feet
Circumference ≈ 9 feet
To calculate the area inside the circular fire pit, we can use the formula:
Area = π × (radius)^2
The radius is half of the diameter, so in this case, the radius is 1.5 feet.
For the exact area, using π as 3.14159:
Area ≈ 3.14159 × (1.5 feet)^2
Area ≈ 7.06858 square feet
For the approximate area, rounding π to 3:
Area ≈ 3 × (1.5 feet)^2
Area ≈ 6.75 square feet
Therefore, the homeowner would need to buy approximately 9 bricks to outline the pit, and approximately 7.07 square feet of sand to cover the space inside the pit (or approximately 6.75 square feet if rounding to a whole number).
Step-by-step explanation:
Can someone help me with thissss
The revenue generated by a company manufacturing lawn mowers is R= -0.5x^2+91x, where x represents the number of mowers produced. The cost to produce the mowers is C= 38x+300,where x is the number of mowers produced.
Find the profit polynomial (Hint: Profit = Revenue - Cost) that describes the total profit generated by the company manufacturing lawn mowers based on the number of mowers produced.
--------------------------
Give the profit earned if 53 mowers are sold. ------------
add work please
The profit function is given as follows:
P(x) = -0.5x² + 53x - 300.
The profit if 53 mowers are sold is given as follows:
P(53) = -$1,598.5. (loss).
How to obtain the profit function?The profit function is obtained as the subtraction of the revenue function by the cost function, as follows:
P(x) = R(x) - C(x).
The functions for this problem are given as follows:
R(x) = -0.5x² + 91x.C(x) = 38x + 300.Hence the profit function is given as follows:
P(x) = -0.5x² + 91x - (38x + 300)
P(x) = -0.5x² + 53x - 300.
The profit if 53 mowers are sold is given as follows:
P(53) = -0.5(53)² + 53(2) - 300
P(53) = -$1,598.5.
More can be learned about profit function at https://brainly.com/question/16866047
#SPJ1
The melody in 12-bar blues often includes which feature, found in some work songs?
bebop
Oragtime
O call & response
Answer:
Hard bop energetic and emotional style of music developed in response to cool jazz and featured fast tempos, loud dynamics, and blues and gospel influences.
Step-by-step explanation:
NO LINKS!! URGENT HELP PLEASE!!
6. Anisha invested $8000 in an account that earns 10% interest. How much money will she have in 15% in the interest is compounded quarterly?
7. Kevin borrowed $32,500 to purchase a new car. If the rate on the loan is 6% compounded annually, how much will he pay in total oer the course of the 5 year loan?
Answer:
6) $35,198.32
7) $43,492.33
Step-by-step explanation:
To solve both these problems, we can use the formula for compound interest:
[tex]\boxed{\begin{minipage}{8.5 cm}\underline{Compound Interest Formula}\\\\$ A=P\left(1+\frac{r}{n}\right)^{nt}$\\\\where:\\\\ \phantom{ww}$\bullet$ $A =$ final amount \\ \phantom{ww}$\bullet$ $P =$ principal amount \\ \phantom{ww}$\bullet$ $r =$ interest rate (in decimal form) \\ \phantom{ww}$\bullet$ $n =$ number of times interest is applied per year \\ \phantom{ww}$\bullet$ $t =$ time (in years) \\ \end{minipage}}[/tex]
Question 6Given values:
P = $8,000r = 10% = 0.1n = 4 (quarterly)t = 15 yearsSubstitute the given values into the formula and solve for A:
[tex]A=8000\left(1+\dfrac{0.1}{4}\right)^{4\cdot 15}[/tex]
[tex]A=8000\left(1+0.025\right)^{60}[/tex]
[tex]A=8000\left(1.025\right)^{60}[/tex]
[tex]A=35198.3179905[/tex]
[tex]A=35198.32[/tex]
Therefore, the balance of Anisha's account after 15 years will be $35,198.32.
[tex]\hrulefill[/tex]
Question 7Given values:
P = $32,500r = 6% = 0.06n = 1 (annually)t = 5 yearsSubstitute the given values into the formula and solve for A:
[tex]A=32500\left(1+\dfrac{0.06}{1}\right)^{1\cdot 5}[/tex]
[tex]A=32500\left(1+0.06\right)^{5}[/tex]
[tex]A=32500\left(1.06\right)^{5}[/tex]
[tex]A=43492.331272[/tex]
[tex]A=43492.33[/tex]
Therefore, Kevin will pay a total of $43,492.33 over the course of the 5 year loan (assuming he doesn't pay any of the loan back over those 5 years).
WILL GIVE BRAINLIEST PLEASE HELP
Answer: line a is parallel to line b
Step-by-step explanation:
the only 2 lines they're talking abt are line a and line b
Suppose that 10,000 barrels of nuclear waste are
Housed in a storage facility. Each barrel has a 1 in
100,000 chance of developing a leak in a given ye
If leaks in different barrels are independent events
what is the probability that at least 1 barrel developing
a leak over a 20 year period
The probability that at least one barrel develops a leak over a 20-year period is approximately 0.1813 or 18.13%.
How to calculate the probabilitySince leaks in different barrels are independent, the probability that none of the 10,000 barrels develops a leak in a given year is:
0.99999¹⁰⁰⁰⁰ = 0.9048
Therefore, the probability that at least one barrel develops a leak in a given year is:
1 - 0.9048 = 0.0952
Now we can use the same approach to find the probability that no barrel develops a leak in 20 years:
0.99999²⁰⁰⁰⁰⁰ = 0.8187
Therefore, the probability that at least one barrel develops a leak over a 20-year period is:
1 - 0.8187 = 0.1813
Learn more about probability on
https://brainly.com/question/24756209
#SPJ1
For which triangle does the Pythagorean Theorem
express the relationship between the lengths of its three
sides?
A
B
C
D
Pythagorean theorem is represented by triangle B.
Which triangle express the relationships according to Pythagorean theorem?
In this question we find four cases of triangle, of which we must determine what triangle can be explained by Pythagorean theorem. This theorem offers a relationship between the three sides of the triangle:
r² = x² + y²
Where:
x, y - Legsr - Hypotenuse.Please notice that hypotenuse is the longest side of the triangle. Graphically speaking, there is a right triangle, that is, a triangle with one right angle. Then, triangle B express the relationship according to Pythagorean theorem.
To learn more on Pythagorean theorem: https://brainly.com/question/20254433
#SPJ1
a metallurgist has one alloy containing 40% aluminum and another containing 61% aluminum. How many pounds of each alloy must he use to make 57 pounds of a third alloy containing 46% aluminum? (round to two decimal places if necessary.)
The number of pounds of 40% alloy and 61% alloy required to make the third alloy containing 46% aluminum is 16.29 pounds and 40.71 pounds respectively.
The number of pounds of alloy that must be used.Let
x = pounds of 40% alloyy = pounds of 61% alloy[tex]\sf x + y = 57[/tex]
[tex]\sf 40x + 61y = 46\times57[/tex]
[tex]\sf x + y = 57[/tex]
[tex]\sf 40x + 61y = 2622[/tex]
Solve simultaneously
From (1)
[tex]\sf x = 57 - y[/tex]
Substitute into (2)
[tex]\sf 40x + 61y = 2622[/tex]
[tex]\sf 40(57-y) + 61y = 2622[/tex]
[tex]\sf 2280 -40y+61y = 2622[/tex]
[tex]\sf -40y+61y = 2622-2280[/tex]
[tex]\sf 21y = 342[/tex]
divide both sides by 21[tex]\sf y = \dfrac{342}{21}[/tex]
[tex]\sf y = 16.29 \ pounds[/tex]
Substitute y = 16.29 pounds into (1)
[tex]\sf x + y = 57[/tex]
[tex]\sf x + 16.29 = 57[/tex]
[tex]\sf x = 57 - 16.29[/tex]
[tex]\sf x = 40.71 \ pounds[/tex]
So therefore, the number of pounds of 40% alloy and 61% alloy is 16.29 pounds and 40.71 pounds respectively.
Read more on simultaneous equation:
brainly.com/question/16863577
John deposited $600 into a savings account at an interest rate of 5% per year. How much interest will he earn in 3 years? What will be his account balance, including interest, after 3 years?
Answer:
$690
Step-by-step explanation:
To find the interest earned in three years, we can use the interest formula
Interest = Principal x Rate x Time
- Principal = $600 (the amount John deposited)
- Rate = 5% per year (as a decimal, this is 0.05)
- Time = 3 years
Interest = $600 x 0.05 x 3 = $90
Therefore, John will earn $90 in interest over 3 years.
To find his account balance, including interest, after 3 years, we can simply add the interest earned to the original principal:
Account balance = Principal + Interest
Account balance = $600 + $90 = $690
Therefore, John's account balance, including interest, after 3 years will be $690.
Answer:
Simple interest: 690, compound interest: 694.5
Step-by-step explanation:
Simple interest = PRT/100 = 600*5*3/100 = 90 = 600+90 = 690
compound interest = P(1+r/100)^n = 600(1+5/100)^3 = 694.5
Your answer depends on if its simple or compound interest if not said so either of the answers I have given to you is right :)
103. In a sample of 50 cars at a local dealership,
there are 12 red cars and 10 cars with backup
cameras. Of the 12 red cars, 4 have backup
cameras. If a car is selected at random from
the given sample, what is the probability
that both of the following are true: the car is
not red and does not have a backup camera?
A. 3/5
B. 16/25
C. 19/25
D. 4/5
Answer:
The correct answer is (D) 4/5.
Step-by-step explanation:
First, we need to find the number of cars that are neither red nor have a backup camera.
Number of cars that are not red = 50 - 12 = 38
Number of cars that do not have a backup camera = 50 - 10 = 40
Number of cars that are neither red nor have a backup camera = 38 + 10 - 50 = 8
Therefore, the probability of selecting a car that is not red and does not have a backup camera is:
P(not red and no backup camera) = 8/50
Simplifying:
P(not red and no backup camera) = 4/25
So the correct answer is (D) 4/5.
Michael records the number of miles he runs each week for nine weeks
6,11,13,8,15,9,11,5,9
Which box plot represents the data
The box plot contains a median of 9, Q1 as 7, and Q3 as 12.
The Smallest: value is 5 and the largest value is 15.
Option B is the correct answer.
We have,
To find the median, we need to arrange the numbers in order:
5, 6, 8, 9, 9, 11, 11, 13, 15
The median is the middle number, which is 9.
To find the quartiles, we first need to find the median of the lower half and upper half of the data.
Lower half: 5, 6, 8, 9, 9
Upper half: 11, 11, 13, 15
To find Q1, we find the median of the lower half:
(6 + 8)/2 = 7
To find Q3, we find the median of the upper half:
(11 + 13)/2 = 12
The smallest number is 5 and the largest number is 15.
Therefore,
Median: 9
Q1: 7
Q3: 12
Smallest: 5
Largest: 15
Learn more about the median here:
https://brainly.com/question/28060453
#SPJ1
ELS, an S corporation, reported a business loss of $1,000,000. Ethan, ELS’s sole shareholder, is involved in ELS’s daily business activities, and he reports $1,200,000 of taxable income from sources other than ELS. What must you know in order to determine how much, if any, of the $1,000,000 loss Ethan may deduct in the current year? How would your answer differ when the CARES Act applied. Explain.
The tax deduction is explained below.
We need to take into account a few things in order to establish how much, if any, of the $1,000,000 loss Ethan may deduct in the current year.
We need to know Ethan's basis in his ELS shares, or shareholder basis. Shareholder basis, which is changed yearly based on numerous criteria such as income, losses, donations, and payouts, symbolizes the investment a shareholder has in the S corporation.
Ethan might be entitled to deduct the loss up to the amount of his basis if his basis is positive.
We must assess if Ethan is covered by the at-risk regulations. The shareholder's amount at risk in the S corporation is the only amount that can be deducted for losses under the at-risk regulations.
If Ethan is subject to these rules, his loss deduction could be limited based on his at-risk amount.
A few tax provisions were temporarily changed as a result of the CARES Act. The CARES Act specifically altered the exclusions for business losses for non-corporate taxpayers, including individual S company shareholders. Ethan might not be able to deduct the $1,000,000 loss because of restrictions on company losses that existed prior to the CARES Act.
The excess business loss limitation for non-corporate taxpayers was temporarily suspended under the CARES Act.
This meant that Ethan and other individual shareholders could write off business losses without being constrained by the prior cap of $250,000 ($500,000 for married taxpayers filing jointly).
Therefore, regardless of any restrictions that could have existed before to the passage of the CARES Act, Ethan would be permitted to deduct the entire $1,000,000 loss in the current year if the CARES Act is applicable.
Taxpayers were temporarily helped by being able to deduct losses from other income. It's crucial to remember that the CARES Act was passed in reaction to the COVID-19 outbreak and that some of its requirements could not be in force at the moment.
Learn more about tax click;
https://brainly.com/question/12611692
#SPJ1
Luke is going to invest $7,600 and leave it in an account for 5 years. Assuming the
interest is compounded daily, what interest rate, to the nearest tenth of a percent,
would be required in order for Luke to end up with $8,900?
The nearest tenth of a percent an interest rate of 4.5% would be required for Luke to end up with $8,900 after 5 years with daily compounding.
The formula for compound interest to solve the problem:
A = [tex]P(1 + r/n)^{(nt)[/tex]
A is the final amount, P is the initial principal, r is the interest rate, n is the number of times the interest is compounded per year and t is the time in years.
P = $7,600
A = $8,900
t = 5 years
n = 365 (compounded daily)
The interest rate r.
Rearrange the formula and solve for r:
r = [tex]n[(A/P)^{(1/(nt))} - 1][/tex]
Substituting the values we know:
r = [tex]365[(8,900/7,600)^{(1/(5 \times 365))} - 1][/tex]
≈ 4.5%
For similar questions on Luke to end up
https://brainly.com/question/20696517
#SPJ11